2021第四届浙江省大学生网络与信息安全竞赛预赛部分Writeup

2021第四届浙江省大学生网络与信息安全竞赛预赛部分Writeup前言:这次比赛感觉比去年难多了,难题都没解出来,还是太菜了orz。WebCheckin纯签到题,题目给了一个网址,直接burpsuite抓包,在响应头上拿到flagREcrackPYC题目提供python字节码,直接找到关键部分进行人工反编译,脚本如下:”””19174SETUP_LOOP48(to224)176LOAD_NAME11(range)…

大家好,又见面了,我是你们的朋友全栈君。

文章目录

Web

Checkin

RE

crackPYC

Crypto

Easy Railfence

MISC

你能找到flag吗?

qrimg


前言:这次预赛感觉比去年难多了,难题都没解出来,还是太菜了orz。

Web

Checkin

纯签到题,题目给了一个网址,直接burpsuite抓包,在响应头上拿到flag  

2021第四届浙江省大学生网络与信息安全竞赛预赛部分Writeup

RE

crackPYC

题目提供python字节码,直接找到关键部分进行人工反编译,脚本如下:

"""
 19         174 SETUP_LOOP              48 (to 224)
            176 LOAD_NAME               11 (range)
            178 LOAD_CONST              36 (32)
            180 CALL_FUNCTION            1
            182 GET_ITER
        >>  184 FOR_ITER                36 (to 222)
            186 STORE_NAME              12 (i)

 20         188 LOAD_NAME               13 (ord)
            190 LOAD_NAME                4 (str)
            192 LOAD_NAME               12 (i)
            
			st[i] = ord(str[i])
			
			194 BINARY_SUBSCR
            196 CALL_FUNCTION            1
            198 LOAD_NAME               10 (key)
            200 LOAD_NAME               12 (i)          
			202 LOAD_NAME                6 (len)
            204 LOAD_NAME               10 (key)
            206 CALL_FUNCTION            1           
			208 BINARY_MODULO
            210 BINARY_SUBSCR
            212 BINARY_XOR
            214 LOAD_NAME                9 (st)
            216 LOAD_NAME               12 (i)
            218 STORE_SUBSCR
			
			st[i] ^ key[i % len(key)] 
			
  2           0 LOAD_CONST               1 (0)
              2 STORE_FAST               1 (num)
			
			 num = 0
  
  3           4 SETUP_LOOP              42 (to 48)
              6 LOAD_GLOBAL              0 (range)
              8 LOAD_CONST               2 (8)
             10 CALL_FUNCTION            1
             12 GET_ITER
        >>   14 FOR_ITER                30 (to 46)
             16 STORE_FAST               2 (i)
			
			 for i in range(0,8)
			
  4          18 LOAD_FAST                1 (num)
             20 LOAD_CONST               3 (7508399208111569251)
             22 BINARY_SUBTRACT
             24 LOAD_CONST               4 (4294967295)
             26 BINARY_MODULO
             28 STORE_FAST               1 (num)
			 
			 num =(num - 7508399208111569251) % 4294967295

  5          30 LOAD_FAST                0 (key)
             32 LOAD_METHOD              1 (append)
             34 LOAD_FAST                1 (num)
             36 LOAD_CONST               5 (24)
             38 BINARY_RSHIFT
             
             key.append(num >> 24)
"""
num = 0
key = []
for i in range(0, 8):
    num = (num - 7508399208111569251) % 4294967295
    key.append(num >> 24)
print(key)
# key = [40, 80, 121, 161, 202, 242, 27, 67]
text = [108, 17, 42, 226, 158, 180, 96, 115, 64, 24, 38, 236, 179, 173, 34, 22, 81, 113, 38, 215, 165, 135, 68, 7, 119,
        97, 45, 254, 250, 172, 43, 62]
for i in range(len(text)):
    tmp = text[i] ^ key[i % 8]
    print(chr(tmp), end="")
# DASCTF{0hH_My_9Uy!_vou_D_1T_0^0}

Crypto

Easy Railfence

题目:

reetdrvhns0eutbftafmeon}linnd=a1cOh!gcedos{neuwkYav0irOceytounw

题目名提示栅栏,直接尝试栅栏编码解密,在12栏发现flag字样

12栏:rtntflag{YOucanc1imbsder0fenceeveny0udOnotevehuandhowitworks!=}

2021第四届浙江省大学生网络与信息安全竞赛预赛部分Writeup

尝试遍历offset无果

改到13栏,遍历offset,最终在offset=5处拿到flag

 2021第四届浙江省大学生网络与信息安全竞赛预赛部分Writeup

flag{YOucanc1imb0verthefenceeveny0udOnotunderstandhowitworks!=}

MISC

你能找到flag吗?

图片给了一张png图片,010Editor打开发现末尾存在压缩包,将压缩包分离。

2021第四届浙江省大学生网络与信息安全竞赛预赛部分Writeup

 得到一个加密压缩包,里面含有大量图片。

2021第四届浙江省大学生网络与信息安全竞赛预赛部分Writeup

 找密码,fuzz一下没什么发现尝试纯数字爆破,十几分钟后,密码10801080

2021第四届浙江省大学生网络与信息安全竞赛预赛部分Writeup

 有密码后直接解压,发现有张图片特别大(没发现特别大,题目其实也给了hint,后面再说)。

2021第四届浙江省大学生网络与信息安全竞赛预赛部分Writeup

010Editor打开,在最后发现一串base64图片编码。

2021第四届浙江省大学生网络与信息安全竞赛预赛部分Writeup

data:image/png;base64,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

截取出base64后运行010Editor内置脚本编码。

2021第四届浙江省大学生网络与信息安全竞赛预赛部分Writeup

2021第四届浙江省大学生网络与信息安全竞赛预赛部分Writeup2021第四届浙江省大学生网络与信息安全竞赛预赛部分Writeup

2021第四届浙江省大学生网络与信息安全竞赛预赛部分Writeup

得到一张图片,fuzz一下宽高不对劲,尝试爆破。

import struct
import binascii
from Crypto.Util.number import bytes_to_long
png_input = open('1.png', 'rb').read()
for i in range(0xFFFF):
stream = png_input[12:20] + struct.pack('>i', i) + png_input[24:29]
crc = binascii.crc32(stream)
if crc == bytes_to_long(png_input[29:33]):
print(i)
# 800

 修改png图片高度为800,出现一串编码,形似镜像后的base64。

2021第四届浙江省大学生网络与信息安全竞赛预赛部分Writeup

解码拿到字符串 Finding…

2021第四届浙江省大学生网络与信息安全竞赛预赛部分Writeup

 用密码去解压flag.rar,拿到最终flag。

DASCTF{flag_1s_close_at_your_hand}

qrimg

下载附件,是个gif文件,将其一帧一帧分离,共计312帧,尝试将其中的图片拖进StegSolve工具查看,发现在Blue plane0时会出现二维码,如下图所示:

2021第四届浙江省大学生网络与信息安全竞赛预赛部分Writeup

312张图片每一张的Blue plane0通道都有二维码,写脚本提取:

from PIL import Image
import pyzbar.pyzbar as pyzbar
import os, base64
def qrcode_parse_content(img):
barcodes = pyzbar.decode(img)
result = []
for barcode in barcodes:
barcode_content = barcode.data.decode('utf-8')
result.append(barcode_content)
return result
def getB0(filepath):
list1 = []
im1 = Image.open(filepath)
for h in range(height):
for w in range(width):
b0 = bin(im1.getpixel((w, h))[2])[-1]  # 取b通道的低0位
list1.append(b0)
return list1
def genpic(list1):
im1 = Image.new("RGB", (width, height), 'white')
i = 0
for y in range(height):
for x in range(width):
if list1[i] == '0':
im1.putpixel([x, y], (0, 0, 0))
else:
im1.putpixel([x, y], (255, 255, 255))
i = i + 1
return im1
if __name__ == "__main__":
tmppath = './tmp/'
if not os.path.exists(tmppath):
os.mkdir(tmppath)
print(tmppath)
tmp = []
for i in range(312):
filepath = './qrimg gif/' + 'IMG' + str(i).zfill(5) + '.bmp'
width, height = Image.open(filepath).size  # 得到宽高
img = genpic(getB0(filepath))
img.save(tmppath + str(i).zfill(5) + '.png')
print(qrcode_parse_content(img))
tmp += qrcode_parse_content(img)
str1 = ''.join(tmp)
print(str1)
while True:
try:
str1 = base64.b64decode(str1).decode("utf-8")
print(150 * '*')
print(str1)
except:
print(str1)
break

最后连成字符串为base64编码,多次解码之后拿到flag

flag{32bb3b8cec39e43a06038a9f96921906}

版权声明:本文内容由互联网用户自发贡献,该文观点仅代表作者本人。本站仅提供信息存储空间服务,不拥有所有权,不承担相关法律责任。如发现本站有涉嫌侵权/违法违规的内容, 请发送邮件至 举报,一经查实,本站将立刻删除。

发布者:全栈程序员-用户IM,转载请注明出处:https://javaforall.cn/158554.html原文链接:https://javaforall.cn

【正版授权,激活自己账号】: Jetbrains全家桶Ide使用,1年售后保障,每天仅需1毛

【官方授权 正版激活】: 官方授权 正版激活 支持Jetbrains家族下所有IDE 使用个人JB账号...

(0)
blank

相关推荐

  • DLL注入与卸载

    DLL注入与卸载DLL注入可用于编写外挂和病毒不易发现。voidCInjectDllToolDlg::StartInject(char*path,intpid){ intpathLen=strlen(path)+sizeof(char);//获取dll目录大小 HANDLEhPro=OpenProcess(PROCESS_ALL_ACCESS,FALSE,pid); if(!

  • html页面根据js名称调用需要的js

    html页面根据js名称调用需要的js

  • java按位取反运算符_二进制按位取反

    java按位取反运算符_二进制按位取反“~”运算符在c、c++、java、c#中都有,要弄懂这个运算符的计算方法,首先必须明白二进制数在内存中的存放形式,二进制数在内存中是以补码的形式存放的。另外正数和负数的补码不一样,正数的补码,反码都是其本身,既:正数9(二进制为:1001)在内存中存储为01001,必须补上符号位(开头的0为符号位)。补码为01001反码为01001,其中前面加的0是符号位,负数的

  • 进程理论基础

    操作系统背景知识顾名思义,进程即正在执行的一个过程。进程是对正在运行程序的一个抽象。进程的概念起源于操作系统,是操作系统最核心的概念,也是操作系统提供的最古老也是最重要的抽象概念之一。操作系统的其

  • oracle导出dmp文件失败_oracle导出数据库dmp文件

    oracle导出dmp文件失败_oracle导出数据库dmp文件–创建用户createuseranhuiidentifiedbyanhui-给予用户权限grantcreatesessiontoanhuigrantconnect,resourcetoanhui;-创建表空间1)先导dmp文件,报错:tablespace‘FMIS_LOB’doesnotexist2)然后创建表空间createtablespaceFMIS_LOB…

  • 2021山东安全员c证考试题库_C类安全员

    2021山东安全员c证考试题库_C类安全员题库来源:安全生产模拟考试一点通公众号小程序2022山东省安全员C证培训试题为山东省安全员C证判断题的新全考试题型!2022年山东省安全员C证考题及答案根据山东省安全员C证考试教材。山东省安全员C证复审模拟考试随时根据安全生产模拟考试一点通上手机同步练习。1、【多选题】一般模板工程通常由()等组成。(ABD)A、面板B、支架C、加固件D、连接件E、螺栓2、【多选题】下列沥青洒布车阀门及油路符合要求的是()。(ABCDE)A、各操作部分应灵活有效,各阀的…

发表回复

您的电子邮箱地址不会被公开。

关注全栈程序员社区公众号